09 September 2014 7 10K Report

Hello all,

I have a set of coupled PDE with well defined domain / boundary conditions, but it's beyond my knowledge to solve it. Could anyone suggest a good method to analytically or numerically solve it? Thanks in advance!

\left[ x \partial_x + y \partial_y + H(x)+ H(y) \right] T_0(x,y) ={}& 1+H(x) T_y(x,y) +H(y) T_x(x,y), \\

\left( 1- \partial_x \right )T_x ={}& T_0,\\

\left( 1- \partial_y \right )T_y ={}& T_0.\\

\\

\\

\text{Domain } \Omega = \left\{ (x,y) \vert x>x_0, y

Similar questions and discussions